Caderno 05 - Matematica

38
Índice Índice Índice Índice Índice MATEMÁTICA (S UMÁRIO) .............................................................. 03 GABARITOS ........................................................................................ 27 ORIENTAÇÕES DE ESTUDO ................................................................ 35 C ADERNO 5 5 5 2013 Ma Ma Ma Ma Matemática temática temática temática temática Atividade III

Transcript of Caderno 05 - Matematica

Page 1: Caderno 05 - Matematica

Índ

ice

Índ

ice

Índ

ice

Índ

ice

Índ

ice

MATEMÁTICA (SUMÁRIO) ..............................................................03

GABARITOS ........................................................................................27

ORIENTAÇÕES DE ESTUDO ................................................................35

CADERNO 55555

2013

MaMaMaMaMatemáticatemáticatemáticatemáticatemáticaAtividade III

Page 2: Caderno 05 - Matematica
Page 3: Caderno 05 - Matematica

Ma

Ma

Ma

Ma

Mate

tic

ate

tic

ate

tic

ate

tic

ate

tic

a SUMÁRIO

POLINÔMIOS

Atividade III ............................................................................... 05

PROBABILIDADE

Atividade III ............................................................................... 07

EQUAÇÕES POLINÔMIAIS

Atividade III ............................................................................... 13

NÚMEROS INTEIROS – DIVISIBILIDADE

Atividade III ............................................................................... 15

MÓDULO DE UM NÚMERO REAL

Atividade III ............................................................................... 17

GEOMETRIA DO ESPAÇO

Atividade III ............................................................................... 19

MATRIZES (2ª PARTE)

Atividade III ............................................................................... 23

TRIGONOMETRIA (3ª PARTE)

Atividade III ............................................................................... 25

Page 4: Caderno 05 - Matematica
Page 5: Caderno 05 - Matematica

Matemática

5

Ensino Médio e Pré-Vestibular/2013 • www.cursoskelvin.com.br

Polinômios

A T I V I D A D E IA T I V I DA D E III

1. (UNIFESP/2007) Se é

verdadeira para todo x real, x ≠ 1, x ≠ 2, então o valor de a ⋅⋅⋅⋅⋅ b é:

a) – 4. d) 2.b) – 3. e) 6.c) – 2.

x

x2 – 3x + 2

ax – 1

bx – 2

= +

2. (FAMECA/2009) Ao dividir p(x) = x100 – 1 por d(x) = x2 – 1

obtém-se o quociente q(x). A soma dos coeficientes de q(x) é:

a) 100. d) 46.

b) 72. e) 36.

c) 50.

3. (UFPR) A respeito do polinômio p(x) = ax3 + bx2 + cx + d,

sendo a, b, c, d números reais, considere as seguintes afirma-

tivas:

I. Se 1 é raiz de p(x), então a + b + c + d = 0.

II. O resto da divisão de p(x) por (x – k) é p(k).

III. Se a = 0, então p(x) tem duas raízes reais.

IV. Se d = 0, então p(x) possui pelo menos uma raiz real.

Assinale a alternativa correta.

a) Somente as afirmativas I, II e IV são verdadeiras.

b) Somente as afirmativas II e IV são verdadeiras.

c) Somente as afirmativas III e IV são verdadeiras.

d) Somente as afirmativas I, II e III são verdadeiras.

e) Somente as afirmativas I e II são verdadeiras

4. (UNESP-INV/2010) Seja a função f(x) = x3 + 2x2 + kx + θ.

Os valores de k e θ, constantes reais, para que 1 + i seja raiz

da função f(x) são, respectivamente:

a) 10 e – 6. d) 0 e 1.

b) 2 e 0. e) – 6 e 8.

c) 1 e 1.

5. (ESPM/2007) Um polinômio que deve ser somado ao

polinômio x3 – 2x2 + 1 para que ele se torne divisível por

x2 + 3 é:

a) –3x + 7. d) – 2x – 5.

b) 3x – 7. e) – 2x + 7.

c) 2x + 5.

6. (ESPM/2006) Um polinômio P(x), quando dividido por

x3 – x deixa como resto o binômio 2x + 1. Se esse polinômio

possui uma única raiz real, ela deve pertencer ao intervalo:

a) ]–2; –1[. d) [ 1/2; 1].

b) [ 0; 1/2 ]. e) [ 1; 2 ].

c) ] –1; 0 ].

7. (FGV) O polinômio P(x) = ax3 + bx2 + cx + 2 satisfaz asseguintes condições:

P(–1) = 0 e P(x) – P(– x) = x3, qualquer que seja x real.

Então:

a) P(1) = – 1. d) P(2) = – 8.b) P(1) = 0. e) P(2) = 12.c) P(2) = 0.

8. (UNESP/2006) Seja z = 1 + i um número complexo.

a) Escreva z e z3 na forma trigonométrica.

b) Determine o polinômio de coeficientes reais, de menor

grau, que tem z e |z|2 como raízes e coeficiente dominan-

te igual a 1.

9. Divida xn + an, n ∈ N*,

a) por x – a, ∀n;

b) por x + a, sendo n ímpar.

10. Divida xn – an, n ∈ N*,a) por x – a, ∀n;b) por x + a, sendo n par.

Page 6: Caderno 05 - Matematica

6

Ensino Médio e Pré-Vestibular/2013 • www.cursoskelvin.com.br

ANOTAÇÕES:

....................................................................................................................

....................................................................................................................

....................................................................................................................

....................................................................................................................

....................................................................................................................

....................................................................................................................

....................................................................................................................

....................................................................................................................

....................................................................................................................

....................................................................................................................

....................................................................................................................

....................................................................................................................

....................................................................................................................

....................................................................................................................

....................................................................................................................

....................................................................................................................

....................................................................................................................

....................................................................................................................

....................................................................................................................

....................................................................................................................

....................................................................................................................

....................................................................................................................

....................................................................................................................

....................................................................................................................

....................................................................................................................

....................................................................................................................

....................................................................................................................

....................................................................................................................

....................................................................................................................

....................................................................................................................

....................................................................................................................

....................................................................................................................

....................................................................................................................

....................................................................................................................

....................................................................................................................

....................................................................................................................

....................................................................................................................

....................................................................................................................

....................................................................................................................

....................................................................................................................

....................................................................................................................

....................................................................................................................

....................................................................................................................

....................................................................................................................

....................................................................................................................

....................................................................................................................

....................................................................................................................

....................................................................................................................

....................................................................................................................

....................................................................................................................

....................................................................................................................

....................................................................................................................

....................................................................................................................

....................................................................................................................

....................................................................................................................

....................................................................................................................

....................................................................................................................

....................................................................................................................

....................................................................................................................

....................................................................................................................

....................................................................................................................

....................................................................................................................

....................................................................................................................

....................................................................................................................

....................................................................................................................

....................................................................................................................

....................................................................................................................

....................................................................................................................

....................................................................................................................

....................................................................................................................

....................................................................................................................

....................................................................................................................

....................................................................................................................

....................................................................................................................

....................................................................................................................

....................................................................................................................

....................................................................................................................

....................................................................................................................

....................................................................................................................

....................................................................................................................

....................................................................................................................

....................................................................................................................

....................................................................................................................

....................................................................................................................

....................................................................................................................

....................................................................................................................

....................................................................................................................

....................................................................................................................

....................................................................................................................

....................................................................................................................

....................................................................................................................

....................................................................................................................

....................................................................................................................

....................................................................................................................

....................................................................................................................

....................................................................................................................

....................................................................................................................

....................................................................................................................

....................................................................................................................

....................................................................................................................

....................................................................................................................

....................................................................................................................

....................................................................................................................

....................................................................................................................

....................................................................................................................

....................................................................................................................

....................................................................................................................

....................................................................................................................

....................................................................................................................

....................................................................................................................

....................................................................................................................

....................................................................................................................

Page 7: Caderno 05 - Matematica

Matemática

7

Ensino Médio e Pré-Vestibular/2013 • www.cursoskelvin.com.br

Probabilidade

A T I V I DA D E III

1. (ENEM/2009) Em um determinado semáforo, as luzes com-pletam um ciclo de verde, amarelo e vermelho em 1 minuto e40 segundos. Desse tempo, 25 segundos são para a luz verde,5 segundos para a amarela e 70 segundos para a vermelha.Ao se aproximar do semáforo, um veículo tem uma determi-nada probabilidade de encontrá-lo na luz verde, amarela ouvermelha. Se essa aproximação for de forma aleatória, pode-se admitir que a probabilidade de encontrá-lo com uma des-sas cores é diretamente proporcional ao tempo em que cadauma delas fica acesa. Suponha que um motorista passa porum semáforo duas vezes ao dia, de maneira aleatória e inde-pendente uma da outra. Qual é a probabilidade de o motoristaencontrar esse semáforo com a luz verde acesa nas duas ve-zes em que passar?a) 1/25. d) 1/3.b) 1/16. e) 1/2.c) 1/9.

2. (PUC/2008) Um marceneiro pintou de azul todas as faces deum bloco maciço de madeira e, em seguida, dividiu-o total-mente em pequenos cubos de 10 cm de aresta. Considerandoque as dimensões do bloco eram 140 cm por 120 cm por90 cm, então a probabilidade de escolher- se aleatoriamenteum dos cubos obtidos após a divisão e nenhuma de suas facesestar pintada de azul é:

a) . d) .13

56

b) . e) .59

89

c) .23

3. (FGV-ADM/2009) Em uma festa de final de ano, dez funcio-nários de uma pequena empresa de consultoria se reúnem paraparticipar do "amigo secreto". Cada um traz um presente queé distribuído, ao acaso, entre os dez.a) Quantas possibilidades há de distribuir os presentes?b) Qual é a probabilidade de certo funcionário receber exa-

tamente o presente que trouxe?

4. (FGV-ECO/2010) Um dado possui seis faces numeradas de1 a 6. As probabilidades de ocorrências das faces com os nú-meros 2, 3, 4, 5 e 6 são, respectivamente, 1/6, 1/12, 1/18,1/27e 1/36. Lançando duas vezes esse dado, a probabilidadede que a soma dos números obtidos em cada lançamentoseja 3 é:a) 1/3. d) 17/81.b) 13/54. e) 1/6.c) 15/69.

5. (FGV-ADM-JUNHO/2009) O espaço amostral de um expe-rimento aleatório é o conjunto dos números inteiros positivosE = {1, 2, 3, ... }. Seja pi a probabilidade de o resultado serigual a i. Suponha que p

i = mi. O valor da expressão é:

a) 1/5.b) 1/6.c) 1/7.d) 1/8.e) impossível de determinar.

6. (FGV-ECO/2009) As seis faces do dado A estão marcadascom 1, 2, 3, 3, 5, 6; e as seis faces do dado B estão marcadascom 1, 2, 4, 4, 5 e 6. Considere que os dados A e B são hones-tos no sentido de que a chance de ocorrência de cada uma desuas faces é a mesma. Se os dados A e B forem lançadossimultaneamente, a probabilidade de que a soma dos núme-ros obtidos seja ímpar é igual a:a) 5/9. d) 1/3.b) 1/2. e) 2/9.c) 4/9.

7. (UNIFESP/2009) Uma urna contém todas as cartelas, do tipoda figura I, totalmente preenchidas com os algarismos 1, 2, 3e 4, de forma que cada linha (horizontal) contempla todos osquatro algarismos.

A probabilidade de se retirar dessa urna, aleatoriamente, umacartela contemplando a configuração da figura II acima, coma exigência adicional de que cada coluna (vertical) e cada umdos subquadrados destacados contenham todos os algarismos(1, 2, 3 e 4) é:

a) . d) .112 ⋅ ⋅ ⋅ ⋅ ⋅ 4! ⋅ ⋅ ⋅ ⋅ ⋅ 4! ⋅ ⋅ ⋅ ⋅ ⋅ 4!

b) . e) .1

16 ⋅ ⋅ ⋅ ⋅ ⋅ 4! ⋅ ⋅ ⋅ ⋅ ⋅ 4! ⋅ ⋅ ⋅ ⋅ ⋅ 4!

c) .1

18 ⋅ ⋅ ⋅ ⋅ ⋅ 4! ⋅ ⋅ ⋅ ⋅ ⋅ 4! ⋅ ⋅ ⋅ ⋅ ⋅ 4!

120 ⋅ ⋅ ⋅ ⋅ ⋅ 4! ⋅ ⋅ ⋅ ⋅ ⋅ 4! ⋅ ⋅ ⋅ ⋅ ⋅ 4!

14! ⋅ ⋅ ⋅ ⋅ ⋅ 4! ⋅ ⋅ ⋅ ⋅ ⋅ 4! ⋅ ⋅ ⋅ ⋅ ⋅ 4!

8. (UNIFESP/2008) Três dados honestos são lançados. A pro-

babilidade de que os três números sorteados possam ser

posicionados para formar progressões aritméticas de razão

1 ou 2 é:

a) 1/36. d) 7/36.

b) 1/6. e) 5/18.

c) 1/9.

Page 8: Caderno 05 - Matematica

8

Ensino Médio e Pré-Vestibular/2013 • www.cursoskelvin.com.br

9. (UNIFESP/2008) Suponha que Moacir esqueceu o númerodo telefone de seu amigo. Ele tem apenas duas fichas, sufici-entes para dois telefonemas.a) Se Moacir só esqueceu os dois últimos dígitos, mas sabe

que a soma desses dois dígitos é 15, encontre o númerode possibilidades para os dois últimos dígitos.

b) Se Moacir só esqueceu o último dígito e decide escolherum dígito ao acaso, encontre a probabilidade de acertaro número do telefone, com as duas tentativas.

10. (UNESP/2008) Astrônomos da Universidade da Califórniafizeram um estudo com cerca de 750 estrelas, sendo 60 delascom planetas e 690 sem planetas (dados aproximados), e cons-tataram que as estrelas com maior índice de ferro (em relaçãoao índice do Sol) têm maior probabilidade de abrigar plane-tas. A tabela mostra o número de estrelas com planetas (C)e sem planetas (S), relativamente ao índice de ferro, denota-do por i.

Utilizando a tabela, mostre que a probabilidadeP(C/{1 ≤ i ≤ 3}), de uma estrela ter planetas dado que 1 ≤ i ≤ 3,é 50% maior que a probabilidade P(C) de uma estrela ter pla-netas.

11. (FUVEST/2009) Um apreciador deseja adquirir, para sua ade-ga, 10 garrafas de vinho de um lote constituído por 4 garrafasda Espanha, 5 garrafas da Itália e 6 garrafas da França, todasde diferentes marcas.a) De quantas maneiras é possível escolher 10 garrafas desse

lote?b) De quantas maneiras é possível escolher 10 garrafas do

lote, sendo 2 garrafas da Espanha, 4 da Itália e 4 daFrança?

c) Qual é a probabilidade de que, escolhidas ao acaso,10 garrafas do lote, haja exatamente 4 garrafas da Itáliae, pelo menos, uma garrafa de cada um dos outros doispaíses?

12. (FUVEST/2008) Em um jogo entre Pedro e José, cada um

deles lança, em cada rodada, um mesmo dado honesto uma

única vez. O dado é cúbico, e cada uma de suas 6 faces es-

tampa um único algarismo de maneira que todos os algaris-

mos de 1 a 6 estejam representados nas faces do dado. Um

participante vence, em uma certa rodada, se a diferença entre

seus pontos e os pontos de seu adversário for, no mínimo, de

duas unidades. Se nenhum dos participantes vencer, passa-se a

uma nova rodada. Dessa forma, determine a probabilidade de:

a) Pedro vencer na primeira rodada.

b) nenhum dos dois participantes vencer na primeira rodada.

c) um dos participantes vencer até a quarta rodada.

13. (FUVEST/2007) Uma urna contém 5 bolas brancas e 3 bolaspretas. Três bolas são retiradas ao acaso, sucessivamente, semreposição. Determine:a) a probabilidade de que tenham sido retiradas 2 bolas pre-

tas e 1 bola branca.b) a probabilidade de que tenham sido retiradas 2 bolas pre-

tas e 1 bola branca, sabendo-se que as três bolas retira-das não são da mesma cor.

14. (UFTM-INVERNO/2010) Um candidato é submetido a umaprova oral, onde o tema a ser desenvolvido é sorteado na hora,e calcula que suas possibilidades de ser aprovado nessa provasão de 3/4, se o tema sorteado for relacionado às matérias queestudou, e de 1/4 se for relacionado a matérias que não estu-dou. O candidato sabe, pela composição do programa, que aprobabilidade de ser sorteado um tema que ele tenha estudadoé 3/5. Nessas condições, pode-se concluir que a probabilidadede que o candidato venha a ser aprovado nessa prova é de:a) 45%. d) 60%.b) 50%. e) 65%.c) 55%.

15. (UFRGS) Uma pessoa tem em sua carteira oito notas de

R$ 1,00, cinco notas de R$ 2,00 e uma nota de R$ 5,00. Se

ela retirar ao acaso três notas da carteira, a probabilidade de

que as três notas retiradas sejam de R$ 1,00 está entre:

a) 15% e 16%. d) 18% e 19%.

b) 16% e 17%. e) 19% e 20%.

c) 17% e 18%.

16. (UFSCar/2009) Os pontos A, B, C, D e E estão dispostos emvértices de triângulos eqüiláteros de lado 2, dispostos em umamalha geométrica, como indicado na figura.

a) Calcule a área do polígono convexo AECBDA.b) Sorteados ao acaso três dos cinco pontos, qual é a proba-

bilidade de que, quando ligados, os pontos sejam vérti-ces de um triângulo de perímetro maior que 10?

Adote: √3 = 1,7 e √7 = 2,6.

17. (UFSCar/2008) Em uma urna foram colocadas cem bolas,numeradas de 1 a 100. Para um sorteio aleatório de uma bola,o jogador A apostou no número 35, o jogador B no número63 e o jogador C no número 72. A, B e C foram os únicosjogadores da partida. Depois de escolhidos os números apos-tados, o organizador do evento divulgou a seguinte regra:

Page 9: Caderno 05 - Matematica

9

Ensino Médio e Pré-Vestibular/2013 • www.cursoskelvin.com.br

Ganhará o prêmio quem acertar o número sorteado e, nãohavendo acertador, ganhará aquele que mais se aproximar donúmero sorteado. Se houver empate entre dois jogadores, ga-nhará aquele que vencer uma partida de cara ou coroa reali-zada com uma moeda honesta.a) Qual é a probabilidade de que A seja o ganhador do prê-

mio?b) Qual é a probabilidade de que B seja o ganhador do prê-

mio?

18. (UFPR/2010) Em uma população de aves, a probabilidade deum animal estar doente é 1/25. Quando uma ave está doente,a probabilidade de ser devorada por predadores é 1/4, e, quan-do não está doente, a probabilidade de ser devorada por pre-dadores é 1/40. Portanto, a probabilidade de uma ave dessapopulação, escolhida aleatoriamente, ser devorada por pre-dadores é de:a) 1,0%. d) 3,4%.b) 2,4%. e) 2,5%.c) 4,0%.

19. (ESPM/2006) A figura abaixo mostra uma parte do mapa deum bairro em que uma pessoa se situa na esquina A.

Se a escolha da direção for aleatória e todas tiverem a mesmachance de ocorrer, depois de ter percorrido exatamente 2 quar-teirões, a probabilidade de que essa pessoa se encontre naesquina B é:a) 1/8. d) 1/9.b) 1/4. e) 1/12.c) 1/16.(Suponha que os quarteirões percorridos sejam distintos)

20. (UFSCar/2007) A probabilidade de que um componente ele-trônico não quebre é chamada de confiabilidade. Para aumen-tar a confiabilidade de um sistema, é comum que se instalemdois componentes eletrônicos de mesma confiabilidade emparalelo. Nesse caso, o sistema só irá falhar se ambos os com-ponentes instalados falharem simultaneamente.

a) Calcule a probabilidade de que um sistema com 2 com-ponentes, cada um de confiabilidade 90%, não falhe.

b) Admita que um sistema com n componentes em paralelosó falhará se os n componentes falharem simultaneamen-

te. Calcule o número de componentes em paralelo quedevem ser instalados em um sistema para que ele tenhaconfiabilidade de 99,9%, sabendo-se que cada compo-nente tem confiabilidade 50%.

Adote: log2 = 0,3

21. (UNICAMP/2009) Três candidatos A, B e C concorrem à pre-sidência de um clube. Uma pesquisa apontou que, dos sóciosentrevistados, 150 não pretendem votar. Dentre os entrevis-tados que estão dispostos a participar da eleição, 40 sóciosvotariam apenas no candidato A, 70 votariam apenas em B, e100 votariam apenas no candidato C. Além disso, 190 disse-ram que não votariam em A, 110 disseram que não votariamem C, e 10 sócios estão na dúvida e podem votar tanto em Acomo em C, mas não em B. Finalmente, a pesquisa revelouque 10 entrevistados votariam em qualquer candidato. Combase nesses dados, pergunta-se:a) Quantos sócios entrevistados estão em dúvida entre vo-

tar em B ou em C, mas não votariam em A? Dentre ossócios consultados que pretendem participar da eleição,quantos não votariam em B?

b) Quantos sócios participaram da pesquisa? Suponha quea pesquisa represente fielmente as intenções de voto detodos os sócios do clube. Escolhendo um sócio ao acaso,qual a probabilidade de que ele vá participar da eleiçãomas ainda não tenha se decidido por um único candidato?

Sugestão: utilize o diagrama de Venn fornecido abaixo.

22. (UFRJ/2006) Em um jogo, cada partida consiste no lança-mento de uma moeda honesta até dez vezes. Se o número decaras obtidas atingir o valor cinco, você perde; caso contrá-rio, você ganha. Calcule a probabilidade de você ganhar umapartida desse jogo.

23. (UFPR/2009) Na central de atendimento ao cliente de umacompanhia telefônica, 60% dos funcionários são do sexo fe-minino. Analisando os relatórios de desempenho de todos osfuncionários que trabalham nessa central (homens e mulhe-res), chegou-se às seguintes conclusões:I. 55% dos problemas relatados pelos clientes são resolvi-

dos na primeira ligação, quando o cliente é atendido poruma funcionária (mulher).

II. 60% dos problemas relatados pelos clientes são resolvi-dos na primeira ligação, quando o cliente é atendido porum funcionário (homem).

Quando se faz uma ligação para essa central de atendimento,o sistema designa, ao acaso, um atendente que tentará resol-ver o problema apresentado pelo cliente.

Page 10: Caderno 05 - Matematica

10

Ensino Médio e Pré-Vestibular/2013 • www.cursoskelvin.com.br

a) Qual a probabilidade de esse atendente resolver o pro-blema do cliente na primeira ligação?

b) Qual é a probabilidade de o atendente ter sido um ho-mem, sabendo que o problema foi resolvido na primeiraligação?

24. (UFPR/2008) Numa sala de aula, há 13 rapazes e 17 moças,sendo que quatro alunos atendem pelo nome de Eduardo etrês alunas atendem pelo nome de Simone.a) De quantos modos diferentes podem ser formados gru-

pos de 5 alunos, sendo 2 rapazes e 3 moças, com a parti-cipação de pelo menos um Eduardo e pelo menos umaSimone?

b) Qual é a probabilidade de um desses grupos ser formadopor exatamente dois Eduardos e três Simones?

25. (ITA/2009) Um determinado concurso é realizado em duasetapas. Ao longo dos últimos anos, 20% dos candidatos doconcurso têm conseguido na primeira etapa nota superior ouigual à nota mínima necessária para poder participar da se-gunda etapa. Se tomarmos 6 candidatos dentre os muitos ins-critos, qual é a probabilidade de no mínimo 4 deles consegui-rem nota para participar da segunda etapa?

26. (ESPM/2006) Seja T o conjunto de todos os triângulos dis-tintos cujos lados possuem medidas inteiras de centímetros esão menores que 5 cm. Escolhendo-se ao acaso um elementode T, a probabilidade de que ele seja um triângulo isóscelesnão equilátero é:a) 2/3.b) 8/13.c) 7/12.d) 6/13.e) 5/12.

27. (UNICAMP/2007) Dois prêmios iguais serão sorteados entredez pessoas, sendo sete mulheres e três homens. Admitindoque uma pessoa não possa ganhar os dois prêmios, respondaàs perguntas abaixo.a) De quantas maneiras diferentes os prêmios podem ser

distribuídos entre as dez pessoas?b) Qual é a probabilidade de que dois homens sejam pre-

miados?c) Qual é a probabilidade de que ao menos uma mulher

receba um prêmio?

28. (UNESP-INV/2009) A proporção de pessoas infectadas porum vírus em uma determinada população é de 1%. O testepara verificar a ocorrência da infecção tem 99% de precisão,isto é, se a pessoa estiver infectada, o teste indica positivo em99% das vezes e negativo em 1%. Da mesma forma, se apessoa não estiver infectada o teste indica negativo em 99%das vezes e positivo em 1%. Qual a probabilidade de um in-divíduo escolhido aleatoriamente nessa população estarinfectado se o teste indicar positivo?

29. (UF-LAVRAS-MG) A fórmula do binômio de Newton é dadapor:

Uma aplicação interessante do binômio de Newton é a depodermos calcular a probabilidade de, em uma família com nfilhos, p deles serem meninos (ou seja, n– p meninas). Essaprobabilidade é dada pela razão entre o termo corresponden-te a p na fórmula acima e a soma total (a + b)n, fazendoa = b = 1. Qual a probabilidade, em uma família com n = 6crianças, de que 3 delas sejam meninos?

+ apbn–p + ... + anb0

np

nn

( a + b)n = a0bn + a1bn–1 + a2bn–2 + ... +

n0

n1

n2

a) . d) .12

b) . e) .38

c) .1564

664

516

30. (FUVEST/2011) Para a prova de um concurso vestibular, fo-ram elaboradas 14 questões, sendo 7 de Português, 4 de Geo-grafia e 3 de Matemática. Diferentes versões da prova poderãoser produzidas, permutando-se livremente essas 14 questões.a) Quantas versões distintas da prova poderão ser produzi-

das?b) A instituição responsável pelo vestibular definiu as ver-

sões classe A da prova como sendo aquelas que seguemo seguinte padrão: as 7 primeiras questões são de Portu-guês, a última deve ser uma questão de Matemática e,ainda mais: duas questões de Matemática não podemaparecer em posições consecutivas. Quantas versões clas-se A distintas da prova poderão ser produzidas?

c) Dado que um candidato vai receber uma prova que co-meça com 7 questões de Português, qual é a probabilida-de de que ele receba uma versão classe A?

31. (UNESP/2012) O mercado automobilístico brasileiro possuivárias marcas de automóveis disponíveis aos consumidores.Para cinco dessas marcas (A, B, C, D e E), a matriz fornece aprobabilidade de um proprietário de um carro de marca dalinha i trocar para o carro de marca da coluna j, quando dacompra de um carro novo. Os termos da diagonal principaldessa matriz fornecem as probabilidades de um proprietáriopermanecer com a mesma marca de carro na compra de umnovo.

Page 11: Caderno 05 - Matematica

11

Ensino Médio e Pré-Vestibular/2013 • www.cursoskelvin.com.br

ANOTAÇÕES:

....................................................................................................................

....................................................................................................................

....................................................................................................................

....................................................................................................................

....................................................................................................................

....................................................................................................................

....................................................................................................................

....................................................................................................................

....................................................................................................................

....................................................................................................................

....................................................................................................................

....................................................................................................................

....................................................................................................................

....................................................................................................................

....................................................................................................................

....................................................................................................................

....................................................................................................................

....................................................................................................................

....................................................................................................................

....................................................................................................................

....................................................................................................................

....................................................................................................................

....................................................................................................................

....................................................................................................................

....................................................................................................................

....................................................................................................................

....................................................................................................................

....................................................................................................................

....................................................................................................................

....................................................................................................................

....................................................................................................................

....................................................................................................................

....................................................................................................................

....................................................................................................................

....................................................................................................................

....................................................................................................................

....................................................................................................................

....................................................................................................................

....................................................................................................................

....................................................................................................................

....................................................................................................................

....................................................................................................................

....................................................................................................................

....................................................................................................................

....................................................................................................................

....................................................................................................................

....................................................................................................................

....................................................................................................................

....................................................................................................................

....................................................................................................................

....................................................................................................................

....................................................................................................................

....................................................................................................................

....................................................................................................................

....................................................................................................................

....................................................................................................................

A probabilidade de um proprietário de um carro da marca Bcomprar um novo carro da marca C, após duas compras, é:

a) 0,25. d) 0,09.b) 0,24. e) 0,00.c) 0,20.

32. (FUVEST/2012)a) Dez meninas e seis meninos participarão de um torneio

de tênis infantil. De quantas maneiras distintas essas 16crianças podem ser separadas nos grupos A, B, C e D,cada um deles com 4 jogadores, sabendo que os gruposA e C serão formados apenas por meninas e o grupo B,apenas por meninos?

b) Acontecida a fase inicial do torneio, a fase semifinal teráos jogos entre Maria e João e entre Marta e José. Osvencedores de cada um dos jogos farão a final. Dado quea probabilidade de um menino ganhar de uma menina é

, calcule a probabilidade de uma menina vencer o tor-

neio.

35

33. (FUVEST/2013) Sócrates e Xantipa enfrentam-se em um po-pular jogo de tabuleiro, que envolve a conquista e ocupaçãode territórios em um mapa. Sócrates ataca jogando três dadose Xantipa se defende com dois. Depois de lançados os dados,que são honestos, Sócrates terá conquistado um território se esomente se as duas condições seguintes forem satisfeitas:

1) o maior valor obtido em seus dados for maior que o maiorvalor obtido por Xantipa;

2) algum outro dado de Sócrates cair com um valor maiorque o menor valor obtido por Xantipa.

a) No caso em que Xantipa tira 5 e 5, qual é a probabilida-de de Sócrates conquistar o território em jogo?

b) No caso em que Xantipa tira 5 e 4, qual é a probabilida-de de Sócrates conquistar o território em jogo?

Page 12: Caderno 05 - Matematica

12

Ensino Médio e Pré-Vestibular/2013 • www.cursoskelvin.com.br

....................................................................................................................

....................................................................................................................

....................................................................................................................

....................................................................................................................

....................................................................................................................

....................................................................................................................

....................................................................................................................

....................................................................................................................

....................................................................................................................

....................................................................................................................

....................................................................................................................

....................................................................................................................

....................................................................................................................

....................................................................................................................

....................................................................................................................

....................................................................................................................

....................................................................................................................

....................................................................................................................

....................................................................................................................

....................................................................................................................

....................................................................................................................

....................................................................................................................

....................................................................................................................

....................................................................................................................

....................................................................................................................

....................................................................................................................

....................................................................................................................

....................................................................................................................

....................................................................................................................

....................................................................................................................

....................................................................................................................

....................................................................................................................

....................................................................................................................

....................................................................................................................

....................................................................................................................

....................................................................................................................

....................................................................................................................

....................................................................................................................

....................................................................................................................

....................................................................................................................

....................................................................................................................

....................................................................................................................

....................................................................................................................

....................................................................................................................

....................................................................................................................

....................................................................................................................

....................................................................................................................

....................................................................................................................

....................................................................................................................

....................................................................................................................

....................................................................................................................

....................................................................................................................

....................................................................................................................

....................................................................................................................

....................................................................................................................

....................................................................................................................

....................................................................................................................

....................................................................................................................

....................................................................................................................

....................................................................................................................

....................................................................................................................

....................................................................................................................

....................................................................................................................

....................................................................................................................

....................................................................................................................

....................................................................................................................

....................................................................................................................

....................................................................................................................

....................................................................................................................

....................................................................................................................

....................................................................................................................

....................................................................................................................

....................................................................................................................

....................................................................................................................

....................................................................................................................

....................................................................................................................

....................................................................................................................

....................................................................................................................

....................................................................................................................

....................................................................................................................

....................................................................................................................

....................................................................................................................

....................................................................................................................

....................................................................................................................

....................................................................................................................

....................................................................................................................

....................................................................................................................

....................................................................................................................

....................................................................................................................

....................................................................................................................

....................................................................................................................

....................................................................................................................

....................................................................................................................

....................................................................................................................

....................................................................................................................

....................................................................................................................

....................................................................................................................

....................................................................................................................

....................................................................................................................

....................................................................................................................

....................................................................................................................

....................................................................................................................

....................................................................................................................

....................................................................................................................

....................................................................................................................

....................................................................................................................

....................................................................................................................

....................................................................................................................

....................................................................................................................

....................................................................................................................

....................................................................................................................

....................................................................................................................

Page 13: Caderno 05 - Matematica

Matemática

13

Ensino Médio e Pré-Vestibular/2013 • www.cursoskelvin.com.br

Equações Polinômiais

A T I V I DA D E III

1. Se m, p e mp são as três raízes reais não nulas da equação

x3 + mx2 + mpx + p = 0, a soma das raízes dessa equação

será:

a) 3. d) 0.

b) 2. e) – 1.

c) 1.

2. (UNICAMP) Considere o polinômio:

P(x) = x3 – 2x2 + 5x + 26.

a) Verifique se o número complexo 2 + 3i é raiz dessepolinômio.

b) Prove que P(x) > 0, para todo real x > – 2.

3. (UNICAMP) Ache todas as raízes (reais e imaginárias) da

equação x6 – 7x3 – 8 = 0.

4. (FUVEST/2008) Um polinômio de grau 3 possui três raízesreais que, colocadas em ordem crescente, formam uma pro-

gressão aritmética em que a soma dos termos é igual a .

A diferença entre o quadrado da maior raiz e o quadrado da

menor raiz é . Sabendo-se que o coeficiente do termo

de maior grau do polinômio é 5, determine:

a) a progressão aritmética.

b) o coeficiente do termo de grau 1 desse polinômio.

95

245

5. (UNESP/2010) As soluções da equação z3 = i, onde z é um

número complexo e i2 = –1, são:

a) z = ± + i e z = – i.√22

12

b) z = ± – i e z = – i.√32

12

c) z = ± + i e z = – i.√32

12

d) z = ± – i e z = – i.√22

12

e) z = ± – i e z = – i.12

√32

6. (FGV)

a) Considere a equação polinomial x3 + x – 5 = 0. Prove

que ela tem uma raiz irracional entre 1 e 2.

b) A equação polinomial x3 – 7x – 6 = 0 tem uma raiz igual

a –1. Encontre as outras raízes.

8. (FGV)a) A equação 2x3 – 8x2 + mx + 16 = 0, sendo m um número

real, tem raízes a, b e c, tais que:a = b + c. Determine o valor de S, tal que S = + + .

b) O polinômio P(x) = 3x4 – 22x3 + 64x2 – 58x + 13 é

divisível por x – . Encontre as raízes da equação

P(x) = 0 no conjunto dos números complexos.

1a

1b

1ac

13

9. (FGV) Dado o polinômio P(x) = x4 + x3 – 6x2 – 4x + k:

a) Resolva a equação P(x) = 0, para k = 8.

b) Determine o valor de k de modo que as raízes estejam

em progressão aritmética de razão igual a 3.

10. (FGV) Considere a função y = f(x), tal que:

f(x) = x3 – 2x2 – x + 2 e cujo gráfico está representado nafigura acima. Determine o conjunto solução da inequação:0 ≤ x3 – 2x2 – x + 14 ≤ 12.

11. (FGV) O polinômio P(x) = x3 + kx2 + 6x + 5 é divisível por

x + 5. Então, a soma das raízes da equação P(x + 1) = 0 é:

a) – 6 . d) – 9.

b) – 7. e) – 3.

c) 6.

12. (UNESP/2010) Uma raiz da equação

x3 – (2a – 1)x2 – a(a + 1)x + 2a2(a – 1) = 0 é (a – 1).

Quais são as outras duas raízes dessa equação?

7. (UFTM-INV/2008) As raízes da equação

x3 + 2008x2 + 2009x + 2010 = 0 são p, q e r.

Então, uma equação que tem como raízes , e , é:

a) 2010x3 + 2009x2 + 2008x + 1 = 0.

b) 2009x3 + 2008x2 + 2010x + 2010 = 0.

c) 2008x3 – 2009x2 + 2010x – 1 = 0.

d) + + + 1 = 0.

e) x3 + + + = 0.

x3

2010x2

2009x

2008

x2

2008x

20091

2010

1p

1q

1r

Page 14: Caderno 05 - Matematica

14

Ensino Médio e Pré-Vestibular/2013 • www.cursoskelvin.com.br

ANOTAÇÕES:

....................................................................................................................

....................................................................................................................

....................................................................................................................

....................................................................................................................

....................................................................................................................

....................................................................................................................

....................................................................................................................

....................................................................................................................

....................................................................................................................

....................................................................................................................

....................................................................................................................

....................................................................................................................

....................................................................................................................

....................................................................................................................

....................................................................................................................

....................................................................................................................

....................................................................................................................

....................................................................................................................

....................................................................................................................

....................................................................................................................

....................................................................................................................

....................................................................................................................

....................................................................................................................

....................................................................................................................

....................................................................................................................

....................................................................................................................

....................................................................................................................

....................................................................................................................

....................................................................................................................

....................................................................................................................

....................................................................................................................

....................................................................................................................

....................................................................................................................

....................................................................................................................

....................................................................................................................

....................................................................................................................

....................................................................................................................

....................................................................................................................

....................................................................................................................

....................................................................................................................

....................................................................................................................

....................................................................................................................

....................................................................................................................

....................................................................................................................

....................................................................................................................

....................................................................................................................

....................................................................................................................

....................................................................................................................

....................................................................................................................

....................................................................................................................

....................................................................................................................

....................................................................................................................

....................................................................................................................

....................................................................................................................

....................................................................................................................

....................................................................................................................

13. (UFBA) Sabendo que a soma das inversas das raízes do

polinômio P(x) = 2x4 + 3x3 – 4x2 + ax + b é igual a 3/2 e que o

resto da divisão de P(x) por x – 2 é igual a 36, determine a e b.

14. (UFMS) Sejam V1 o volume de um cubo de aresta x, e V

2 o

volume de um paralelepípedo retângulo cuja área da base é11x – 38 e cuja altura é igual a x. Encontre o maior valor de xtal que V

1 = V

2 + 40.

Page 15: Caderno 05 - Matematica

Matemática

15

Ensino Médio e Pré-Vestibular/2013 • www.cursoskelvin.com.br

Números Inteiros – Divisibilidade

A T I V I DA D E III

1. (FUVEST) O produto de dois números positivos, que nãosão primos entre si, é igual a 825. Então o máximo divisorcomum desses dois números é:a) 1. d) 11.b) 3. e) 15.c) 5.

2. (FUVEST) Um estudante terminou um trabalho que tinha npáginas. Para numerar todas essas páginas, iniciando com apágina 1, ele escreveu 270 algarismos. Então o valor de n é:a) 99. d) 148.b) 112. e) 270.c) 126.

7. (UNIFESP) Entre os primeiros mil números inteiros positi-vos, quantos são divisíveis pelos números 2, 3, 4 e 5?a) 60. d) 16.b) 30. e) 15.c) 20.

6. (FUVEST) A diferença entre dois números inteiros positivosé 10. Ao multiplicar um pelo outro, um estudante cometeuum engano, tendo diminuído em 4 o algarismo das dezenasdo produto. Para conferir seus cálculos, dividiu o resultadoobtido pelo menor dos fatores, obtendo 39 como quociente e22 como resto. Determine os dois números.

5. (FUVEST) Um número inteiro positivo n de 4 algarismosdecimais satisfaz às seguintes condições:I. a soma dos quadrados dos 1º e 4º algarismos é 58;II. a soma dos quadrados dos 2º e 3º algarismos é 52;III. se deste número n subtrairmos o número 3816, obtere-

mos um número formado pelos mesmos algarismos donúmero n, mas na ordem contrária.Qual é esse número?

4. (UNIFESP) Um número inteiro n, quando dividido por 7, deixaresto 5. Qual será o resto na divisão de n2 + n por 7?a) 5.b) 4.c) 3.d) 2.e) 1.

3. (PUC-SP) Resolver um criptograma aritmético significa usara estratégia “tentativa e erro” para determinar quais núme-ros satisfazem as condições de um dado problema. Considereo criptograma seguinte, em que cada letra representa apenasum único algarismo, não nulo.

(AR)2 = BAR

Para os valores de A, R e B encontrados, é correto afirmarque o “número” BARRA está compreendido entre:

a) 45000 e 50000. d) 60000 e 65000.b) 50000 e 55000. e) 65000 e 70000.c) 55000 e 60000.

10. (UFScar-SP/2008) Considere a, b e c algarismos que fazemcom que a conta a seguir, realizada com números de três al-garismos, esteja correta.

Nas condições dadas, b ⋅ c–a é igual a:

a) 0. d) 1.

b) . e) 16.

c) .

116

4 a 51 5 bc 7 7

14

8. (UFTM) Considere a seguinte operação de soma,A + B + BB = CCC, onde A, B e C são os algarismos dosrespectivos números, são diferentes entre si e estão contidosno conjunto {1, 2, 3,4, 5, 6, 7, 8, 9}.Então a soma (A + B + C) é:a) 11. d) 14.b) 12. e) 15.c) 13.

9. (PUC-SP) Para a orientação dos maquinistas, ao longo deuma ferrovia existem placas com a indicação da quilometra-gem. Um trem percorre essa ferrovia em velocidade constan-te e, num dado instante, seu maquinista observa uma placaem que o número indicador da quilometragem tinha 2 alga-rismos. Após 30 minutos, ele passa por uma outra em que,curiosamente, os algarismos assinalados eram os mesmos daprimeira, só que escritos na ordem inversa. Decorridos30 minutos de sua passagem pela segunda placa, ele passapor uma terceira em que o número marcado tinha os mesmosalgarismos das anteriores mas na mesma ordem dos da pri-meira e com um zero intercalado entre eles. Nessas condi-ções, a velocidade desse trem, em quilômetros por hora, era

a) 72. d) 116.b) 90. e) 120.c) 100.

11. (UNICAMP) Um número inteiro positivo de três algarismosterminam em 7. Se este último algarismo for colocado antesdos outros dois, o novo número assim formado excede de21 o dobro do número original. Qual é o número inicial?Justifique sua resposta.

Page 16: Caderno 05 - Matematica

16

Ensino Médio e Pré-Vestibular/2013 • www.cursoskelvin.com.br

12. (UNESP-2012) Em um programa de platéia da TV brasileira,cinco participantes foram escolhidos pelo apresentador paratentarem acertar o número de bolas de gude contidas em umaurna de vidro transparente. Aquele que acertasse ou mais seaproximasse do número real de bolas de gude contidas naurna ganharia um prêmio. Os participantes A, B, C, D e Edisseram haver, respectivamente, 1195, 1184, 1177, 1250 e1232 bolas na urna. Sabe-se que nenhum dos participantesacertou o número real de bolas, mas que um deles se enganouem 30 bolas, outro em 25, outro em 7, outro em 48 e, final-mente, outro em 18 bolas. Podemos concluir que quem ga-nhou o prêmio foi o participante:a) A. d) D.b) B. e) E.c) C.

ANOTAÇÕES:

....................................................................................................................

....................................................................................................................

....................................................................................................................

....................................................................................................................

....................................................................................................................

....................................................................................................................

....................................................................................................................

....................................................................................................................

....................................................................................................................

....................................................................................................................

....................................................................................................................

....................................................................................................................

....................................................................................................................

....................................................................................................................

....................................................................................................................

....................................................................................................................

....................................................................................................................

....................................................................................................................

....................................................................................................................

....................................................................................................................

....................................................................................................................

....................................................................................................................

....................................................................................................................

....................................................................................................................

....................................................................................................................

....................................................................................................................

....................................................................................................................

....................................................................................................................

....................................................................................................................

....................................................................................................................

....................................................................................................................

....................................................................................................................

....................................................................................................................

....................................................................................................................

....................................................................................................................

....................................................................................................................

....................................................................................................................

....................................................................................................................

....................................................................................................................

....................................................................................................................

....................................................................................................................

....................................................................................................................

....................................................................................................................

....................................................................................................................

....................................................................................................................

....................................................................................................................

....................................................................................................................

....................................................................................................................

....................................................................................................................

....................................................................................................................

....................................................................................................................

....................................................................................................................

....................................................................................................................

....................................................................................................................

....................................................................................................................

....................................................................................................................

13. (UNICID-INV/2013) Um mágico pediu para uma pessoa daplatéia escolher um número natural qualquer e deu as seguin-tes instruções:

• Multiplicar o número escolhido por 2.• Somar 3 ao resultado.• Multiplicar por 5 o resultado.• Subtrair do resultado o número escolhido.• Escolher um dos dígitos diferentes de zero do número

obtido e escrevê-lo em um papel, de modo que o mágiconão visse.

• Falar os demais dígitos em qualquer ordem.

A pessoa disse: “1, 0, 2, 2 e 3” e o mágico pôde constatar queo dígito escrito no papel era

a) 3.b) 5.c) 7.d) 1.e) 9.

Page 17: Caderno 05 - Matematica

Matemática

17

Ensino Médio e Pré-Vestibular/2013 • www.cursoskelvin.com.br

Módulo de um Número Real

A T I V I DA D E III

1. (FEI-SP) Resolver a desigualdade (em R):

52|m|

17|m|

+ < 1.

2. (KELVIN/2012) Resolva no universo dos reais a inequação:| log

2 x | < 1.

4. (FGV-ADM-JUNHO/2009) No plano cartesiano, a área daregião determinada pelas inequações simultâneas x2 + y2 ≤ 1e | x + y | ≥ 1 é igual a:

a) – . d) – 1.

b) – . e) – .

c) – .

π2

14

π2

12

π2

34

π2

π2

54

5. (UNESP-INV/2012) No conjunto IR dos números reais, oconjunto solução S da inequação modular | x | ⋅⋅⋅⋅⋅ | x – 5 | ≥ 6 é:

a) S = {x ∈ R / –1 ≤ x ≤ 6}.

b) S = {x ∈ R / x ≤ –1 ou 2 ≤ x ≤ 3}.

c) S = {x ∈ R / x ≤ –1 ou 2 ≤ x ≤ 3 ou x ≥ 6}.

d) S = {x ∈ R / x ≤ 2 ou x ≥ 3}.

e) S = R.

6. Construa o gráfico da função real f dada por

f(x) = | 2x + 3 | – 2 .

7. Sejam as funções reais f e g definidas por

f(x) = | x | e g(x) = , x ≠ – .

Encontre o conjunto solução da equação f(g(x) = 3.

x – 12x + 3

32

3. (FUVEST/2012) Determine para quais valores reais de x éverdadeira a desigualdade | x2 – 10x + 21 | ≤ | 3x – 15 |.

8. (FUVEST/2007)a) Represente, no sistema de coordenadas desenhado na fo-

lha de respostas, os gráficos das funções

f(x) = | 4 – x2 | e g(x) =

b) Resolva a inequação

| 4 – x2 | ≤

x + 72

x + 72

9. (INSPER) Considere a função f dada pela lei f(x) = || x | – 4|.a) Esboce o gráfico da função f no sistema cartesiano for-

necido abaixo.b) Sendo a um número real, determine todos os valores de

a para os quais a equação f(x) = a ⋅⋅⋅⋅⋅ | x | possui exatamen-te quatro soluções reais.

Page 18: Caderno 05 - Matematica

18

Ensino Médio e Pré-Vestibular/2013 • www.cursoskelvin.com.br

ANOTAÇÕES:

....................................................................................................................

....................................................................................................................

....................................................................................................................

....................................................................................................................

....................................................................................................................

....................................................................................................................

....................................................................................................................

....................................................................................................................

....................................................................................................................

....................................................................................................................

....................................................................................................................

....................................................................................................................

....................................................................................................................

....................................................................................................................

....................................................................................................................

....................................................................................................................

....................................................................................................................

....................................................................................................................

....................................................................................................................

....................................................................................................................

....................................................................................................................

....................................................................................................................

....................................................................................................................

....................................................................................................................

....................................................................................................................

....................................................................................................................

....................................................................................................................

....................................................................................................................

....................................................................................................................

....................................................................................................................

....................................................................................................................

....................................................................................................................

....................................................................................................................

....................................................................................................................

....................................................................................................................

....................................................................................................................

....................................................................................................................

....................................................................................................................

....................................................................................................................

....................................................................................................................

....................................................................................................................

....................................................................................................................

....................................................................................................................

....................................................................................................................

....................................................................................................................

....................................................................................................................

....................................................................................................................

....................................................................................................................

....................................................................................................................

....................................................................................................................

....................................................................................................................

....................................................................................................................

....................................................................................................................

....................................................................................................................

....................................................................................................................

....................................................................................................................

....................................................................................................................

....................................................................................................................

....................................................................................................................

....................................................................................................................

....................................................................................................................

....................................................................................................................

....................................................................................................................

....................................................................................................................

....................................................................................................................

....................................................................................................................

....................................................................................................................

....................................................................................................................

....................................................................................................................

....................................................................................................................

....................................................................................................................

....................................................................................................................

....................................................................................................................

....................................................................................................................

....................................................................................................................

....................................................................................................................

....................................................................................................................

....................................................................................................................

....................................................................................................................

....................................................................................................................

....................................................................................................................

....................................................................................................................

....................................................................................................................

....................................................................................................................

....................................................................................................................

....................................................................................................................

....................................................................................................................

....................................................................................................................

....................................................................................................................

....................................................................................................................

....................................................................................................................

....................................................................................................................

....................................................................................................................

....................................................................................................................

....................................................................................................................

....................................................................................................................

....................................................................................................................

....................................................................................................................

....................................................................................................................

....................................................................................................................

....................................................................................................................

....................................................................................................................

....................................................................................................................

....................................................................................................................

....................................................................................................................

....................................................................................................................

....................................................................................................................

....................................................................................................................

....................................................................................................................

....................................................................................................................

....................................................................................................................

....................................................................................................................

Page 19: Caderno 05 - Matematica

Matemática

19

Ensino Médio e Pré-Vestibular/2013 • www.cursoskelvin.com.br

Geometria do Espaço

A T I V I DA D E III

1. (UFF) Considere duas superfícies S = ABCD e S’ = E’B’C’obtidas, respectivamente, pelas interseções de um cilindrocircular reto e de uma semi-esfera com semi-planos que for-mam um ângulo diedro de 60°, conforme as figuras abaixo.

Tem-se:O - centro da base do cilindroOE - altura do cilindroOB - raio da base do cilindroO’E’ - raio da semi-esferaOE = OB = O’E’

Sendo área(S) a área da superfície S e área(S’) a área da su-

perfície S’, calcule o valor de .área(S)área(S’)

2. (UFF) Na figura a seguir vê-se um cubo cujas arestas medem1 metro. M, N, P, Q, R e S representam, respectivamente, ospontos médios de suas arestas AB, BC, HC, DG, GF e AH.

Calcule:a) A área do triângulo MRS.b) O volume do sólido MNPQRS indicado na figura.

3. (FUVEST/2011) A esfera ε, de centro O e raio r > 0, é tan-gente ao plano α. O plano β é paralelo a α e contém O. Nes-sas condições, o volume da pirâmide que tem como base umhexágono regular inscrito na intersecção de ε com β e, comovértice, um ponto em α, é igual a:

a) . d) .

b) . e) .

c) .

√3r3

4

5√3r3

16

3√3r3

8

7√3r3

16

√3r3

2

4. (FUVEST) O ângulo θ formado por dois planos α e β é tal

que tgθ = . O ponto P pertence a α e a distância de P a β

vale 1. Então, a distância de P à reta intersecção de α e β éigual a:

a) √3. d) √7.

b) √5. e) √8.

c) √6.

√55

5. (UEM-PR) Uma das possíveis moléculas do fulereno apre-senta-se na forma de 12 faces pentagonais e 20 faces hexago-nais, sendo que cada uma de suas faces apresenta átomos decarbono nos vértices. A figura abaixo ilustra a estruturatridimensional dessa forma do fulereno.

Sobre essa molécula, assinale a alternativa correta.a) Ela apresenta mais de 100 ligações carbono-carbono.b) O número de átomos de carbono é 60.c) A forma da molécula desse fulereno é de um poliedro

regular.d) Ela não pode se considerada umas das formas alotrópicas do

carbono porque cada carbono forma apenas três ligações.e) Os ângulos das faces são congruentes.

6. (UNICAMP) Uma caixa d’água tem o formato de um troncode pirâmide de bases quadradas e paralelas, como mostra afigura abaixo, na qual são apresentadas as medidas referentesao interior da caixa.a) Qual o volume total da caixa d’água?

b) Se a caixa contém m3 de água, a que altura de sua

base está o nível d’água?

1316

7. (UNICAMP/2012) Um brilhante é um diamante com umalapidação particular, que torna essa gema a mais apreciadadentre todas as pedras preciosas.a) Em gemologia, um quilate é uma medida de massa, que

corresponde a 200 mg. Considerando que a massa espe-cífica do diamante é de aproximadamente 3,5 g/cm3, de-termine o volume de um brilhante com 0,7 quilate.

b) A figura a seguir apresenta a seção transversal de umbrilhante. Como é muito difícil calcular o volume exatoda pedra lapidada, podemos aproximá-lo pela soma dovolume de um tronco de cone (parte superior) com o deum cone (parte inferior).

Page 20: Caderno 05 - Matematica

20

Ensino Médio e Pré-Vestibular/2013 • www.cursoskelvin.com.br

Determine, nesse caso, o volume aproximado do brilhante.

Dica: o volume de um tronco de cone pode ser obtido em-pregando-se a fórmula

V = h (R2 + rR + r2) , em que R e r são os raios das

bases e h é a altura do tronco.

π3

8. (ITA) Considere um cilindro circular reto, de volume igual a360πcm3, e uma pirâmide regular cuja base hexagonal estáinscrita na base do cilindro. Sabendo que a altura da pirâmideé o dobro da altura do cilindro e que a área da base da pirâmi-de é de cm2, então, a área lateral da pirâmide mede, em cm2.

a) 18√427. d) 108√3.

b) 27√427. e) 45√427.

c) 36√427.

9. (UERJ) Duas esferas metálicas maciças de raios iguais a 8cm e 5 cm são colocadas, simultaneamente, no interior de umrecipiente de vidro com forma cilíndrica e diâmetro da basemedindo 18 cm. Neste recipiente despeja-se a menor quanti-dade possível de água para que as esferas fiquem totalmentesubmersas, como mostra a figura.

Posteriormente, as esferas são retiradas do recipiente.A altura da água, em cm, após a retirada das esferas, corres-ponde, aproximadamente, a:

a) 10,6. c) 14,5.b) 12,4. d) 25,0.

10. (UERJ) Uma cuba de superfície semi-esférica, com diâmetrode 8 cm, está fixada sobre uma mesa plana. Uma bola degude de forma esférica, com raio igual a 1 cm, encontra-sesob essa cuba.

Desprezando a espessura do material usado para fabricar acuba, determine:

a) a maior área, em cm2, pela qual a bola de gude poderá sedeslocar na superfície da mesa;

b) o volume, em cm3, da maior esfera que poderia ser colo-cada embaixo dessa cuba.

11. (ITA) Um dos catetos de um triângulo retângulo mede√2 cm. O volume do sólido gerado pela rotação deste triân-gulo em torno da hipotenusa é πcm3. Determine os ângulosdeste triângulo.

12. (UERJ) Um cilindro circular reto é inscrito em um cone, demodo que os eixos desses dois sólidos sejam colineares, con-forme representado na ilustração abaixo.

A altura do cone e o diâmetro da sua base medem, cada um,12 cm. Admita que as medidas, em centímetros, da altura edo raio do cilindro variem no intervalo ] 0;12 [ de modo queele permaneça inscrito nesse cone. Calcule a medida que aaltura do cilindro deve ter para que sua área lateral seja má-xima.

13. (FUVEST) Considere uma caixa sem tampa com a forma deum paralelepípedo reto de altura 8 m e base quadrada de lado6 m. Apoiada na base, encontra-se uma pirâmide sólida retade altura 8 m e base quadrada com abaixo 6 m. O espaçointerior à caixa e exterior à pirâmide é preenchido com água,até uma altura h, a partir da base (h ≤ 8).

Determine o volume da água para um valor arbitrário de h,0 ≤ h ≤ 8.

14. (FUVEST) A figura abaixo mostra uma pirâmide reta de basequadrangular ABCD de lado 1 e altura EF = 1. Sendo G oponto médio da altura EF e a α medida do ângulo AGB, entãocos a vale:

a) 1/2.

b) 1/3.

c) 1/4.

d) 1/5.

e) 1/6.

^

Page 21: Caderno 05 - Matematica

21

Ensino Médio e Pré-Vestibular/2013 • www.cursoskelvin.com.br

15. (PUC/2010) Num plano cartesiano ortogonal, seja o triângu-lo ABC, em que A, B e C são as interseções das retas deequações:

Considerando que a unidade das medidas nos eixos coorde-nados é o metro e π = 3,14, então a rotação do triângulo ABCem torno do eixo das ordenadas gera um recipiente cuja ca-pacidade, em litros, é um númeroa) menor que 15000.b) compreendido entre 15000 e 18000.c) compreendido entre 18000 e 21000.d) compreendido entre 21000 e 24000.e) maior que 24000.

y = – x – 1, y = x – 1 e y = 232

32

16. (PUC) Considere o quadrilátero que se obtém unindo quatrodas intersecções das retas de equações x = 0, y = 0, y = 6 e3x – y – 6 = 0 e suponha que uma xícara tem o formato dosólido gerado pela rotação desse quadrilátero em torno doeixo das ordenadas. Assim sendo, qual o volume do café naxícara no nível da metade de sua altura?

a) 31π.

b) 29π.

c) 24π.

d) 21π.

e) 19π.

17. (PUC/2007) Considere o triângulo isósceles ABC, tal queAB = BC = 10 cm e CA = 12 cm. A rotação desse triânguloem torno de um eixo que contém o lado AB gera um sólidocujo volume, em centímetros cúbicos, é:

a) 256π.

b) 298,6π.

c) 307,2π.

d) 316π.

e) 328,4π.

18. (INSPER-2007) Se R > 0, sabe-se que, no plano cartesiano, a

relação (x – a)2 + (y – b)2 ≤ R2 representa um círculo de cen-

tro (a, b) e raio R. De modo análogo, no espaço euclidiano

tridimensional, a relação (x – a)2 + (y – b)2 + (z – c)2 ≤ R2

representa uma esfera de centro (a, b, c) e raio R. Dado um

sistema de coordenadas (Oxyz) no espaço, considere a esfera

(ε) dada pela relação (x – 2)2 + (y – 3)2 + (z – 4)2 ≤ 36.

A interseção de ε com o plano que contém os eixos Ox e Oy

é um círculo de área:

a) 10π.

b) 16π.

c) 20π.

d) 24π.

e) 36π.

19. (PUC/2013) Calvin, por natureza, é um menino maldoso e“arteiro”. A tira abaixo mostra a “engenhoca” que ele cons-truiu para perturbar o sossego de seu pai. Ele espera que, aoser aberta a porta, a água existente no balde escorra pelacanaleta e molhe seu pai!

Sabe-se que o balde tem a forma de um tronco de cone de16 cm de altura e raios das bases de medidas 11 cm e 8 cm; a

água em seu interior ocupa de sua capacidade. Assim

sendo, quantos litros de água Calvin pretende jogar no seupai? (Considere a aproximação: π = 3)

a) 2,965. d) 2,894.b) 2,912. e) 2,890.c) 2,904.

23

20. (UNESP/2013) Para confeccionar um porta-jóias a partir deum cubo maciço e homogêneo de madeira com 10 cm de ares-ta, um marceneiro dividiu o cubo ao meio, paralelamente àsduas faces horizontais. De cada paralelepípedo resultanteextraiu uma semiesfera de 4 cm de raio, de modo que seuscentros ficassem localizados no cruzamento das diagonais daface de corte, conforme mostra a sequência de figuras.

Sabendo que a densidade da madeira utilizada na confecçãodo porta-jóias era de 0,85 g/cm3 e admitindo π ≈ 3, a massaaproximada do porta-jóias, em gramas, é:a) 636.b) 634.c) 630.d) 632.e) 638.

Page 22: Caderno 05 - Matematica

22

Ensino Médio e Pré-Vestibular/2013 • www.cursoskelvin.com.br

ANOTAÇÕES:

....................................................................................................................

....................................................................................................................

....................................................................................................................

....................................................................................................................

....................................................................................................................

....................................................................................................................

....................................................................................................................

....................................................................................................................

....................................................................................................................

....................................................................................................................

....................................................................................................................

....................................................................................................................

....................................................................................................................

....................................................................................................................

....................................................................................................................

....................................................................................................................

....................................................................................................................

....................................................................................................................

....................................................................................................................

....................................................................................................................

....................................................................................................................

....................................................................................................................

....................................................................................................................

....................................................................................................................

....................................................................................................................

....................................................................................................................

....................................................................................................................

....................................................................................................................

....................................................................................................................

....................................................................................................................

....................................................................................................................

....................................................................................................................

....................................................................................................................

....................................................................................................................

....................................................................................................................

....................................................................................................................

....................................................................................................................

....................................................................................................................

....................................................................................................................

....................................................................................................................

....................................................................................................................

....................................................................................................................

....................................................................................................................

....................................................................................................................

....................................................................................................................

....................................................................................................................

....................................................................................................................

....................................................................................................................

....................................................................................................................

....................................................................................................................

....................................................................................................................

....................................................................................................................

....................................................................................................................

....................................................................................................................

....................................................................................................................

....................................................................................................................

21. (UNICAMP/2013) Um recipiente cúbico de aresta a e semtampa, apoiado em um plano horizontal, contém água até a

altura a. Inclina-se lentamente o cubo, girando-o em um

ângulo θ em torno de uma das arestas da base, como estárepresentado na figura abaixo.

a) Supondo que o giro é interrompido exatamente antes dea água começar a derramar, determine a tangente do ân-gulo θ.

b) Considerando, agora, a inclinação tal que tg(θ) = ,

com 0 < θ < , calcule o valor numérico da expressão

cos(2θ) – sen(2θ).

34

14

π2

Page 23: Caderno 05 - Matematica

Matemática

23

Ensino Médio e Pré-Vestibular/2013 • www.cursoskelvin.com.br

Matrizes (2ª parte)

A T I V I DA D E III

1. (UNICAMP) Uma matriz real quadrada P é dita ortogonal se

PT = P–1, ou seja, se sua transposta é igual a sua inversa.

a) Considere a matriz P abaixo. Determine os valores de

a e b para que P seja ortogonal.

Dica: você pode usar o fato de que P–1P = I, em que I é

a matriz identidade.

b) Uma certa matriz A pode ser escrita na forma A = QR,

sendo Q e R as matrizes abaixo. Sabendo que Q é

ortogonal, determine a solução do sistema Ax = b, para o

vetor b dado, sem obter explicitamente a matriz A.

Dica: lembre-se de que x = A–1b.

–1/3 –2/3 –2/3–2/3 a –1/3–2/3 b 2/3

P =

Page 24: Caderno 05 - Matematica

24

Ensino Médio e Pré-Vestibular/2013 • www.cursoskelvin.com.br

ANOTAÇÕES:

....................................................................................................................

....................................................................................................................

....................................................................................................................

....................................................................................................................

....................................................................................................................

....................................................................................................................

....................................................................................................................

....................................................................................................................

....................................................................................................................

....................................................................................................................

....................................................................................................................

....................................................................................................................

....................................................................................................................

....................................................................................................................

....................................................................................................................

....................................................................................................................

....................................................................................................................

....................................................................................................................

....................................................................................................................

....................................................................................................................

....................................................................................................................

....................................................................................................................

....................................................................................................................

....................................................................................................................

....................................................................................................................

....................................................................................................................

....................................................................................................................

....................................................................................................................

....................................................................................................................

....................................................................................................................

....................................................................................................................

....................................................................................................................

....................................................................................................................

....................................................................................................................

....................................................................................................................

....................................................................................................................

....................................................................................................................

....................................................................................................................

....................................................................................................................

....................................................................................................................

....................................................................................................................

....................................................................................................................

....................................................................................................................

....................................................................................................................

....................................................................................................................

....................................................................................................................

....................................................................................................................

....................................................................................................................

....................................................................................................................

....................................................................................................................

....................................................................................................................

....................................................................................................................

....................................................................................................................

....................................................................................................................

....................................................................................................................

....................................................................................................................

....................................................................................................................

....................................................................................................................

....................................................................................................................

....................................................................................................................

....................................................................................................................

....................................................................................................................

....................................................................................................................

....................................................................................................................

....................................................................................................................

....................................................................................................................

....................................................................................................................

....................................................................................................................

....................................................................................................................

....................................................................................................................

....................................................................................................................

....................................................................................................................

....................................................................................................................

....................................................................................................................

....................................................................................................................

....................................................................................................................

....................................................................................................................

....................................................................................................................

....................................................................................................................

....................................................................................................................

....................................................................................................................

....................................................................................................................

....................................................................................................................

....................................................................................................................

....................................................................................................................

....................................................................................................................

....................................................................................................................

....................................................................................................................

....................................................................................................................

....................................................................................................................

....................................................................................................................

....................................................................................................................

....................................................................................................................

....................................................................................................................

....................................................................................................................

....................................................................................................................

....................................................................................................................

....................................................................................................................

....................................................................................................................

....................................................................................................................

....................................................................................................................

....................................................................................................................

....................................................................................................................

....................................................................................................................

....................................................................................................................

....................................................................................................................

....................................................................................................................

....................................................................................................................

....................................................................................................................

....................................................................................................................

....................................................................................................................

....................................................................................................................

Page 25: Caderno 05 - Matematica

Matemática

25

Ensino Médio e Pré-Vestibular/2013 • www.cursoskelvin.com.br

Trigonometria (3ª parte)

A T I V I DA D E III

1. (ITA) A respeito da solução da equação senx + √3 cosx = 2,

0 ≤ x < 2π, podemos afirmar que:

a) existe apenas uma solução no 1º quadrante.

b) existe apenas uma solução no 2º quadrante.

c) existe apenas uma solução no 3º quadrante.

d) existe apenas uma solução no 4º quadrante.

e) existe duas soluções no intervalo 0 ≤ x < 2π.

2. (FUVEST)

a) Expresse sen3α em função de senα.

b) Resolva e inequação sen3α > 2 senα para 0 < α < π.

4. Resolva a inequação senx ≥ cosx no intervalo 0 ≤ x ≤ 2π.

3. Resolva a equação 2senx | senx | + 3senx = 2.

5. Resolva as inequações:

a) | senx | ≤

b) | cosx | >√32

12

6. Que valores de x, com 0 ≤ x ≤ 2π , verificam a desigualdade

√3senx – cosx ≤1?

7. Calcule tg arc sen

√53

8. Calcule cos arc cos + arc cos7

251525

9. (INSPER) Uma nova loteria foi inventada, a “trigoloteria”.Nessa loteria, um apostador precisa escolher apenas um nú-

mero θ no intervalo . Depois disso, o apostador gira

duas roletas A e B ao mesmo tempo duas vezes. Os resultadosde cada roleta podem ser apenas “prêmio” ou “perdeu”. Oapostador ganha na trigoloteria se obtiver duas vezes conse-cutivas o resultado “prêmio” em pelo menos uma das roletas.A tabela abaixo indica as probabilidades das duas roletas for-necerem o resultado “prêmio”em cada uma das duas rodadas.

π2

0,

10. (INSPER) Considere a expressão y = cos(2x), em que x ∈ R,para responder o que se pede a seguir.

a) Determine o menor valor real de x para o qual cos(2x) = 1.

Dados: log2 ≈ 0,30 e logπ ≈ 0,50.

b) Sabendo que cos(2x) = , calcule cos(2x + 1).34

a) Calcule, em função de θ, a probabilidade de uma pessoaque aposta uma vez não ganhar na trigoloteria.

b) Determine o valor de θ para o qual a probabilidade dealguém que faz uma aposta não ganhar na trigoloteria éa menor possível.

Page 26: Caderno 05 - Matematica

26

Ensino Médio e Pré-Vestibular/2013 • www.cursoskelvin.com.br

ANOTAÇÕES:

....................................................................................................................

....................................................................................................................

....................................................................................................................

....................................................................................................................

....................................................................................................................

....................................................................................................................

....................................................................................................................

....................................................................................................................

....................................................................................................................

....................................................................................................................

....................................................................................................................

....................................................................................................................

....................................................................................................................

....................................................................................................................

....................................................................................................................

....................................................................................................................

....................................................................................................................

....................................................................................................................

....................................................................................................................

....................................................................................................................

....................................................................................................................

....................................................................................................................

....................................................................................................................

....................................................................................................................

....................................................................................................................

....................................................................................................................

....................................................................................................................

....................................................................................................................

....................................................................................................................

....................................................................................................................

....................................................................................................................

....................................................................................................................

....................................................................................................................

....................................................................................................................

....................................................................................................................

....................................................................................................................

....................................................................................................................

....................................................................................................................

....................................................................................................................

....................................................................................................................

....................................................................................................................

....................................................................................................................

....................................................................................................................

....................................................................................................................

....................................................................................................................

....................................................................................................................

....................................................................................................................

....................................................................................................................

....................................................................................................................

....................................................................................................................

....................................................................................................................

....................................................................................................................

....................................................................................................................

....................................................................................................................

....................................................................................................................

....................................................................................................................

....................................................................................................................

....................................................................................................................

....................................................................................................................

....................................................................................................................

....................................................................................................................

....................................................................................................................

....................................................................................................................

....................................................................................................................

....................................................................................................................

....................................................................................................................

....................................................................................................................

....................................................................................................................

....................................................................................................................

....................................................................................................................

....................................................................................................................

....................................................................................................................

....................................................................................................................

....................................................................................................................

....................................................................................................................

....................................................................................................................

....................................................................................................................

....................................................................................................................

....................................................................................................................

....................................................................................................................

....................................................................................................................

....................................................................................................................

....................................................................................................................

....................................................................................................................

....................................................................................................................

....................................................................................................................

....................................................................................................................

....................................................................................................................

....................................................................................................................

....................................................................................................................

....................................................................................................................

....................................................................................................................

....................................................................................................................

....................................................................................................................

....................................................................................................................

....................................................................................................................

....................................................................................................................

....................................................................................................................

....................................................................................................................

....................................................................................................................

....................................................................................................................

....................................................................................................................

....................................................................................................................

....................................................................................................................

....................................................................................................................

....................................................................................................................

....................................................................................................................

....................................................................................................................

....................................................................................................................

....................................................................................................................

....................................................................................................................

....................................................................................................................

Page 27: Caderno 05 - Matematica

AT I V I D A D E S

I I I

Ga

Ga

Ga

Ga

Ga

ba

rito

sb

arito

sb

arito

sb

arito

sb

arito

s

• M A T E M Á T I C A •

Page 28: Caderno 05 - Matematica
Page 29: Caderno 05 - Matematica

Matemática

29

• GABARITO S •

RE S P O S TA S DA S AT I V I D A D E S III

Polinômios ATIVIDADE III

1. Alternativa C 2. Alternativa C

3. Alternativa A 4. Alternativa E

5. Alternativa B 6. Alternativa C

7. Alternativa C

z3 = 2√2 cos + isen3π4

3π4

8. a) |z| = |1 + i|

|z| = √12 + 12 ∴ |z| = √2

z = 1 + i

+ i1

√2

1

√2z = √2

z = √2 cos + isenπ4

π4

z3 = √2 cos + isenπ4

π4

3

z3 = (√2)3 cos + isen3π4

3π4

Resposta:

z = √2 cos + isen eπ4

π4

z3 = 2√2 cos + isen3π4

3π4

b) |z|2 = (√2)2 ∴∴∴∴∴ |z|2 = 2

Se 1 + i é raiz de um polinômio de coeficientes reais, então 1 – itambém o é. O polinômio de coeficientes reais, de menor grau, quetem 1 + i e 2 como raízes e coeficiente dominante igual a 1 é dadopor p(x) = [x – (1 + i)] [x – (1 – i)] [x – 2].

Temos:

p(x) = [(x – 1) – i] [(x – 1) + i] [x – 2]

p(x) = [(x – 1)2 – i2] [x – 2]

p(x) = (x2 – 2x + 2) (x – 2)

p(x) = x3 – 4x2 + 6x – 4

Resposta: x3 – 4x2 + 6x – 4

9. a) Q(x) = xn – 1 + axn – 2 + a2xn – 3 + ... + an – 1 e R(x) = 2an

b) Q(x) = xn – 1 – axn – 2 + a2xn – 3 – ... + an – 1 e R(x) = 0

10. a) Q(x) = xn – 1 + axn – 2 + a2xn – 3 + ... + an – 1 e R(x) = 0

b) Q(x) = xn – 1 – axn – 2 + a2xn – 3 – ... – an – 1 e R(x) = 0

Probabilidade ATIVIDADE III

A probabilidade de encontrar o semáforo duas vezes com a luz verde

é: 116

14

2

=

P = =25

25 + 5 + 7014

1. Alternativa B

A probabilidade de o motorista encontrar o semáforo com a luz verde édada por

2. Alternativa B

4. Alternativa D

6. Alternativa A

A soma dos resultados é ímpar se, e somente se, um resultado é par e

o outro é ímpar.

A probabilidade de obter um resultado par com o dado A é 2/6.

A probabilidade de obter um resultado ímpar com o dado B é 2/6.

A probabilidade de obter um resultado par com A e um ímpar com B é

2/6 ⋅⋅⋅⋅⋅ 2/6 = 4/36.

A probabilidade de obter um resultado ímpar com o dado A é 4/6.

A probabilidade de obter um resultado par com o dado B é 4/6.

A probabilidade de obter um resultado ímpar com A e um par com B é

4/6 ⋅⋅⋅⋅⋅ 4/6 = 16/36.

A probabilidade de a soma dos resultados ser ímpar é

4/36 + 16/36 = 20/36, ou seja, 5/9.

7. Alternativa A

Temos 4! modos de dispor esses algarismos na primeira linha e 4! nasegunda e 4! na terceira e 4! na quarta.

Assim, o número de casos possíveis é 4! ⋅⋅⋅⋅⋅ 4! ⋅⋅⋅⋅⋅ 4! ⋅⋅⋅⋅⋅ 4!.

Temos exatamente duas maneiras de atender à exigência adicional:

Logo, a probabilidade é: P = =2

4! ⋅⋅⋅⋅⋅ 4! ⋅⋅⋅⋅⋅ 4! ⋅⋅⋅⋅⋅ 4!

1

12 ⋅⋅⋅⋅⋅ 4! ⋅⋅⋅⋅⋅ 4! ⋅⋅⋅⋅⋅ 4!

8. Alternativa C

5. Alternativa D

Considerando E = {1, 2, 3, ..., i, ...}, temos:

p1 + p2 + p3 + ... + pi + ... = 1

m + m2 + m3 + ... + m i + ... = 1 ∴∴∴∴∴ = 1m

1 – m

∴∴∴∴∴ m =12

Assim, temos:

pi = 1 – (p1 + p2 + p3) = 1 – + + = 1 – =∞Σ

i = 4

12

14

18

78

18

3. a) Os presentes podem ser distribuídos de 10! maneiras.

Assim, 10! = 3628800 possibilidades.

Resposta: 3628800

b) Seja o “senhor X” esse certo funcionário.

Das 10! maneiras possíveis de se distribuírem os presentes, o “se-nhor X” recebe o seu próprio presente em 9! delas. Assim, a proba-bilidade pedida é:

9!10!

=1

10

Resposta: 110

Page 30: Caderno 05 - Matematica

30

• GABARITO S •

RESPOSTA DA A T IV IDADE I I I

12. Representando os resultados de Pedro e José respectivamente, porum par ordenado, temos:

a) Para Pedro vencer na primeira rodada, temos as seguintes possi-bilidades:

(3; 1); (4; 1); (5; 1); (6; 1); (4; 2); (5; 2); (6; 2) (5; 3); (6; 3) ou (6; 4)

Assim, a probabilidade é dada por: , ou seja,1036

518

Resposta: 518

b) Para nenhum dos participantes vencer na primeira rodada, deve-mos ter:

(1; 1); (2; 2); (3; 3); (4; 4); (5; 5); (6; 6)

(2; 1); (1; 2); (3; 2); (2; 3); (4; 3); (3; 4)

(5; 4); (4; 5); (6; 5) ou (5; 6)

Assim, a probabilidade é dada por: , ou seja,1636

49

Resposta: 49

11. a) O número de maneiras de escolher 10 garrafas desse lote é:

C15,10 = = 300315!

10! ⋅ ⋅ ⋅ ⋅ ⋅ 5!

Resposta: 3003 maneiras

b) Do enunciado, o número de maneiras é:

C4,2 ⋅⋅⋅⋅⋅ C5,4 ⋅⋅⋅⋅⋅ C6,4 = ⋅⋅⋅⋅⋅ ⋅⋅⋅⋅⋅ = 4504!

2! ⋅ ⋅ ⋅ ⋅ ⋅ 2!

5!

4! ⋅ ⋅ ⋅ ⋅ ⋅ 1!

6!

4! ⋅ ⋅ ⋅ ⋅ ⋅ 2!

Resposta: 450 maneiras

c) Com 4 garrafas da Itália, para que haja pelo menor uma garrafa decada um dos outros países no lote, só não pode haver nesse loteas 6 garrafas da França. Assim o número de casos favoráveis é:

C5,4 ⋅⋅⋅⋅⋅ C10,6 – C6,6 = 5 ⋅⋅⋅⋅⋅ [210 – 1] = 1045.

A probabilidade pedida é: =

Resposta:

95

273

1045

300395

273

9

10

8

9

8

10

9. a) 4

b) 1/5

a) Dado que a e b são dígitos de 0 a 9, tais que a + b = 15, temos asseguintes possibilidades para (a, b): (6, 9), (7, 8), (8, 7) e (9, 6).

Há, portanto, 4 possibilidades para os dígitos.

b) A probabilidade de ele errar nas duas tentativas é

p = ⋅⋅⋅⋅⋅ = .

Logo, a probabilidade de ele acertar com as duas tentativas é dada

por p = 1 – p = 1 – = =8

10

2

10

1

510. Do enunciado, temos:

P(C / {1 ≤ i ≤ 3}) = = e P(C) = =30 + 15

300 + 75

45

375

60

750

30

375Dividindo, temos:

P(C / {1 ≤ i ≤ 3})

P(C)= = 1,5

45

30

Ou seja, P(C / {1 ≤ i ≤ 3}) é 50% maior que P(C).

13. a) O número de elementos do espaço amostral é: C8,3 = = 56

O número de elementos do evento ‘‘saírem 2 bolas pretas e 1 bolabranca’’ é C3,2 ⋅⋅⋅⋅⋅ C5,1 = 3 ⋅⋅⋅⋅⋅ 5 = 15

A probabilidade é igual a:

Resposta:

b) Do enunciado o ‘‘novo’’ espaço amostral é:

C8,3 – C5,3 – C3,3 = 56 – 10 – 1 = 45.

Como o evento é o mesmo do item anterior, a probabilidade pedida

é: P = =

Resposta:

8!3! ⋅⋅⋅⋅⋅ 5!

1556

1556

1545

13

13

14. Alternativa C

P(A) = P(aprovado com um tema que estudou) = 3/4

P(B) = P(aprovado com um tema que não estudou) = 1/4

P(E) = P(ser sorteado um tema que tenha estudado) = 3/5

P(E) = P(ser sorteado um tema que não tenha estudado) = 1– 3/5 = 2/5

Do enunciado, temos:

A probabilidade de ser aprovado, é:

Ser sorteado um tema que tenha estudado e ser aprovado com essetema ou ser sorteado um tema que não tenha estudado e ser aprova-do com esse tema

P(aprovado) = P(E) ⋅⋅⋅⋅⋅ P(A) + P(E) ⋅⋅⋅⋅⋅ P(B) = ⋅ ⋅ ⋅ ⋅ ⋅ + ⋅ ⋅ ⋅ ⋅ ⋅ = = 0,55

ou seja 55%.

35

34

25

14

1120

16. a) 5√3

b) 4/9

17. a) Como = 49, A ganhará se o número sorteado for de

1 a 48, e disputará na moeda com B se o número for 49.

A probabilidade pedida é:

P = + ⋅⋅⋅⋅⋅ = 48,5%

35 + 632

48100

12

1100

Resposta: 48,5%

b) Como = 67,5, B ganhará se o número sorteado for de

50 a 67, e disputará na moeda com A se o número for 49.

A probabilidade pedida é:

P = + ⋅⋅⋅⋅⋅ = 18,5%

63 + 722

18100

12

1100

Resposta: 18,5%

18. Alternativa D

19. Alternativa E

c) Para que um dos participantes vença até a quarta rodada, temosas seguintes possibilidades:

• alguém vence na primeira rodada: 1 – =

• ninguém vence na primeira e alguém vence na segunda

rodada: ⋅⋅⋅⋅⋅

• ninguém vence nem na primeira nem na segunda, e alguém ven-

ce na terceira rodada: ⋅⋅⋅⋅⋅ ⋅⋅⋅⋅⋅

• ninguém vence na primeira, nem na segunda, nem na terceira, e

alguém vence na quarta rodada: ⋅⋅⋅⋅⋅ ⋅⋅⋅⋅⋅ ⋅⋅⋅⋅⋅

49

59

49

59

49

49

59

49

49

49

59

+++++⋅⋅⋅⋅⋅ 1=====

+++++59

⋅⋅⋅⋅⋅49

+++++59

49

2

⋅⋅⋅⋅⋅ +++++59

49

3

⋅⋅⋅⋅⋅ 59

59

49

+++++ 1681

+++++ 64729

⋅⋅⋅⋅⋅ = = = = ====== 59

1261729

63056561

Resposta: 63056561

Assim, temos:

15. Alternativa A

Page 31: Caderno 05 - Matematica

RESPOSTA DA A T IV IDADE I I I

31

• GABARITO S •

20. A probabilidade de que o sistema não falhe é igual a 1 menos a proba-bilidade de ele falhar.

Assim:

a) Com 2 componentes, temos:

P = 1 – ⋅ ⋅ ⋅ ⋅ ⋅ = 1 – = 99%

Resposta: 99%

110

110

1100

Falhar e Falhar

b) Do enunciado, temos:

1 – = ∴ ∴ ∴ ∴ ∴ 1 – =

50100

n99,9100

12

n9991000

∴∴∴∴∴ n ⋅⋅⋅⋅⋅ log2 = 3 ∴∴∴∴∴ n = = 103

0,3

∴∴∴∴∴ = ∴ ∴ ∴ ∴ ∴ 2n = 103 ∴ ∴ ∴ ∴ ∴ log2n = log103

12

n1

1000

Resposta: 10

22. 193/512

23. a) 57%

b) ≈ 42%2456

24. a) 13272 b) P = =

a) Outra solução pode ser obtida calculando o total de grupos possí-veis (sem a restrição de Eduardos ou Simones) e subtraindo destetotal os grupos que não têm Eduardos ou que não tem Simones,assim:

Total de grupos (sem restrições): C13,2 ⋅⋅⋅⋅⋅ C17,3 = 53040

Grupos sem Eduardos: C9,2 ⋅⋅⋅⋅⋅ C17,3 = 24480

Grupos sem Simones: C13,2 ⋅⋅⋅⋅⋅ C14,3 = 28392

Grupos sem Eduardos e sem Simones: C9,2 ⋅⋅⋅⋅⋅ C14,3 = 13104Logo o número procurado é:N = 53040 – (24480 + 28392 – 13104) = 13272 ⇒ N = 13272

613272

12212

P = =

b) Pela tabela acima, há exatamente 6 grupos diferentes formadospor exatamente dois Eduardos e três Simones, como o total degrupos, logo a probabilidade procurada é:

613272

12212

21. Do enunciado, temos:

Assim:

a) • 20 sócios devem votar em B ou em C, mas não votariam em A.

• 40 + 10 + 100 = 150 sócios não votariam em B.

Resposta: 20 e 150

b) • 10 + 0 + 10 + 20 + 40 + 70 + 100 + 150 = 400 sócios participamda pesquisa.

• A probabilidade pedida é: = =

Resposta: 400 e

10 + 10 + 20 + 0400

40100

110

110

Portanto, se o teste indica positivo, a probabilidade de ele estar infecta-

do é = , ou seja, 50%.

Resposta: 50%

0,99 ⋅⋅⋅⋅⋅ 0,010,99 ⋅⋅⋅⋅⋅ 0,01 + 0,01 ⋅⋅⋅⋅⋅ 0,99

12

28. O teste indica positivo e

ele está infectado: 0,99 ⋅⋅⋅⋅⋅ 0,01

ele não está infectado: 0,01 ⋅⋅⋅⋅⋅ 0,99

27. a) 45 maneiras b) c)115

1415

25.

26. Alternativa B

533125

Equações Polinômiais ATIVIDADE III

1. Alternativa D

2. a) Sim, pois P(2 + 3i) = 0

b) Como P(x) = (x + 2)(x2 – 4x +13)

x2 – 4x +13 > 0, ∀ x ∈ R pois ∆ < 0

concluímos que P(x) > 0 se x + 2 > 0, então x > – 2

3. S = –1, 2, –1 + √3 ⋅⋅⋅⋅⋅ i, –1 – √3 ⋅⋅⋅⋅⋅ i, ,1 + √3 ⋅⋅⋅⋅⋅ i

21 – √3 ⋅⋅⋅⋅⋅ i

2

4. a) Representando as raízes por x1, x2 e x3, com x1 < x2 < x3, temos

x1 = x2 – r e x3 = x2 + r, em que r é a razão da progressão aritmé-

tica (x1, x2, x3). Temos:

x1 + x2 + x3 =95

x2 – r + x2 + x2 + r =95

3x2 + ∴ ∴ ∴ ∴ ∴ x

2 =

95

35

+ + r2 – – + r2 =

925

6r5

925

6r5

245

= ∴ ∴ ∴ ∴ ∴ r = 212r5

245

De x1 = x2 – r, temos x1 = – 2 = .35

–75

De x3 = x2 + r, temos x3 = + 2 = .35

135

Resposta: –75

35

135

, ,

De x23 – x2

1 = , temos:245

235

+ r – – r =

235

245

29. Alternativa E

30. a) Podemos permutar as 14 questões de 14! modos diferentes.

Resposta: 14! versões distintas.

b) Temos 7! possibilidades de colocar as 7 primeiras de Português, edepois 3 possibilidades para a última questão (Matemática), e de-pois 4 possibilidades para a penúltima (Geografia). Resta aindaposicionarmos 5 questões, 2 de Matemática e 3 de Geografia, nãopodendo as de Matemática ocupar posições consecutivas. Paraisso, podemos permutar todas as 5 e descontar os casos em queas 2 de Matemática ocupam posições consecutivas.Assim: 5! - 4! ⋅ ⋅ ⋅ ⋅ ⋅ 2! = 72Pelo Princípio Fundamental da Contagem, temos, então:

Resposta: 864 ⋅⋅⋅⋅⋅ 7! versões distintas.

c) O número de casos possíveis em que o candidato recebe umaprova que começa com 7 questões de Português é 7! ⋅⋅⋅⋅⋅ 7!. Assim, aprobabilidade P pedida é:

Resposta: 6/35.864 ⋅⋅⋅⋅⋅ 7!7! ⋅ ⋅ ⋅ ⋅ ⋅ 7!

P = =635

31. Alternativa D

32. a) 47250

b) 44125

33. a)

b)

227

43216

Page 32: Caderno 05 - Matematica

2. S =

32

• GABARITO S •

RESPOSTA DA A T IV IDADE I I I

Sendo P(t) = t3 + 6t2 + 6 t + 5 e sendo t1, t2 e t3 as raízes de P(t) = 0,

temos t1 + t2 + t3 = –6 (Girard).

Sendo t = x + 1 e x1, x2 e x3 as raízes da equação P(x + 1) = 0, temos

x1 + 1 + x2 + 1 + x3 + 1 = – 6 e, portanto, x1 + x2 + x3 = –9.

12. 2a e – a

14. a) a = –2, b = –2, c = 8 e as raízes de p(x) são 2, –2, 1 + i e 1 – i

b) q(x) = k(x – 1)(x + 3)(x2 + 1); com k ∈ R*.

1. Alternativa CSejam x e y os dois números inteiros positivos e d, o seu máximo divisorcomum. Podemos afirmar que existem números inteiros a e b tais quex = d ⋅ ⋅ ⋅ ⋅ ⋅ a e y = d ⋅ ⋅ ⋅ ⋅ ⋅ b.

b) Podemos representar o polinômio por 5x3 + bx2 + cx + d, em que b,

c e d são constantes a determinar.

Sendo as raízes x1, x2 e x3, temos x1 ⋅⋅⋅⋅⋅ x2 + x1 ⋅⋅⋅⋅⋅ x3 + x2 ⋅⋅⋅⋅⋅ x3 = .c5

Do item anterior, podemos afirmar que

⋅ ⋅ ⋅ ⋅ ⋅ + ⋅ ⋅ ⋅ ⋅ ⋅ + ⋅ ⋅ ⋅ ⋅ ⋅ =135

35

–75

c5

–75

35

135

–7325

c5

= ∴ ∴ ∴ ∴ ∴ c =–73

5

Resposta: –735

5. Alternativa C

6. a) Sendo p(x) = x3 + x – 5, temos p(1) = –3 e p(2) = 5.

Como p(1) e p(2) têm sinais opostos, podemos afirmar, pelo teorema

de Bolzano, que a equação x3 + x – 5 = 0 admite uma raiz real r,

com 1 < r < 2. (1)

Como o número 5 não possui divisor r, com 1 < r < 2, podemos

afirmar, pelo teorema das raízes racionais, que r não é um número

racional. (2)

De (1) e (2), podemos afirmar que a equação x3 + x – 5 = 0 tem

uma raiz irracional entre 1 e 2 (c.q.d).

b) Como –1 é raiz, x3 – 7x – 6 é divisível por x + 1. Efetuemos essa

divisão pelo dispositivo de Briot-Ruffini:

Logo, x3 – 7x – 6 ≡ (x + 1) (x2 – x – 6).

Para encontrar as outras raízes, resolvemos a equação x2 – x – 6 = 0.

Essas raízes são os números –2 e 3.

Resposta: –2 e 3.

1 0 –7 – 6

–1 1 –1 – 6 0

7. Alternativa A

8. a) –

b) , 1, 3 + 2i e 3 – 2i

12

13

9. a) {1, – 2, 2}

b) Não existe um valor da constante k de modo que as raízes estejam

em progressão aritmética de razão igual a 3.

10. {x ∈ R: – 2 ≤ x ≤ – 1 ou 1 ≤ x ≤ 2}

11. Alternativa D

Vamos admitir que k seja uma constante.

Da divisibiliade de P(x) por x + 5, temos:

P(–5) = 0

(– 5)3 + k ⋅⋅⋅⋅⋅ (–5)2 + 6 (– 5) + 5 = 0

–125 + 25 k – 30 + 5 = 0

25 k = 150 ∴∴∴∴∴ k = 6 e P(x) = x3 + 6x2 + 6x + 5

Números inteiros – Divisibilidade ATIVIDADE III

Logo x ⋅⋅⋅⋅⋅ y = d2 ⋅⋅⋅⋅⋅ a ⋅⋅⋅⋅⋅ b (I)

Do enunciado, temos que x ⋅ ⋅ ⋅ ⋅ ⋅ y = 825 = 3 ⋅ ⋅ ⋅ ⋅ ⋅ 52 ⋅ 11 (II)

Como x e y não são primos entre si, temos que d ≠ 1e, portanto, pode-mos concluir, de (I) e (II), que d = 5.

2. Alternativa C 3. Alternativa D

4. Alternativa D 5. 7463

6. 41 e 31

7. Alternativa DOs números devem ser múltiplos de 2, 3, 4 e 5.O mmc (2, 3, 4, 5) é igual a 60. Do algoritmo da divisão em IN temos1000 = 16 ⋅⋅⋅⋅⋅ 60 + 40. Assim, existem 16 múltiplos de 60 de 1 a 1000.

8. Alternativa C 9. Alternativa B

10. Alternativa D 11. 357

12. Alternativa E 13. Alternativa C

Módulo de um número real ATIVIDADE III

1. S = m ∈ R : m < – ou m >

374

374

12

0,

3. 1 ≤ x ≤ 4 ou 6 ≤ x ≤ 9.

4. Alternativa D

5. Alternativa C

6.

7. S = – 2; – 87

8. a)

b) x ∈ IR : – ≤ x ≤ –1 ou ≤ x ≤ 352

12

9. a)

b) 0 < a < 1.

Page 33: Caderno 05 - Matematica

33

• GABARITO S •

RESPOSTA DA A T IV IDADE I I I

Matrizes (2ª parte) ATIVIDADE III

b) x =

1

1– 4

1. a) a = e b = –23

13

9. a) P(θ) = 1 –

b)

sen(2θ)2

π4

2

10. a) 2,67

b) 18

ANOTAÇÕES:

....................................................................................................................

....................................................................................................................

....................................................................................................................

....................................................................................................................

....................................................................................................................

....................................................................................................................

....................................................................................................................

....................................................................................................................

....................................................................................................................

....................................................................................................................

....................................................................................................................

....................................................................................................................

....................................................................................................................

....................................................................................................................

....................................................................................................................

....................................................................................................................

....................................................................................................................

....................................................................................................................

....................................................................................................................

....................................................................................................................

....................................................................................................................

....................................................................................................................

....................................................................................................................

....................................................................................................................

....................................................................................................................

....................................................................................................................

....................................................................................................................

....................................................................................................................

....................................................................................................................

....................................................................................................................

....................................................................................................................

....................................................................................................................

....................................................................................................................

....................................................................................................................

....................................................................................................................

....................................................................................................................

....................................................................................................................

....................................................................................................................

....................................................................................................................

....................................................................................................................

....................................................................................................................

....................................................................................................................

....................................................................................................................

....................................................................................................................

b)

Geometria do Espaço ATIVIDADE III

1. Resposta: 1

2. a) m2 b) m314

√24

3. Alternativa E 4. Alternativa C

5. Alternativa B

6. a) m3 b) 2 m214

7. a) 0,04 cm3 b) 3,8πmm3

8. Alternativa A 9. Alternativa C

10. a) 8π cm2 b) cm332π3

11. 30°, 60° e 90°

12. Resposta: 6 cm

13. V(h) = 36h – 96 + (8 – h)3316

14. Alternativa B 15. Alternativa A

16. Alternativa E 17. Alternativa C

18. Alternativa C 19. Alternativa B

20. Alternativa D

21. a) 12

717

Trigonometria (3ª parte) ATIVIDADE III

1. Alternativa A

5. a) S = x ∈ R/2kπ ≤ x ≤ + 2kπ ou + 2kπ ≤ x ≤ +

+ 2kπ ou + 2kπ ≤ x ≤ (k + 1)2π, k ∈ Z

π6

5π6

7π6

11π6

4. ;π4

5π4

3. S = x ∈ R/x = + 2kπ ou x = + 2kπ, k ∈ Zπ6

5π6

2. a) sen3α = 3senα – 4sen3α

b) S = 0; ∪ ; π5π6

π6

b) S = x ∈ R/2kπ < x < + 2kπ ou + 2kπ < x < +

+ 2kπ ou + 2kπ < x < (k + 1)2π, k ∈ Z

π6

5π6

7π6

11π6

6. S = 0; ∪ [π; 2π ]π3

7. √52

8. – 35

Page 34: Caderno 05 - Matematica

34

Ensino Médio e Pré-Vestibular/2013 • www.cursoskelvin.com.br

....................................................................................................................

....................................................................................................................

....................................................................................................................

....................................................................................................................

....................................................................................................................

....................................................................................................................

....................................................................................................................

....................................................................................................................

....................................................................................................................

....................................................................................................................

....................................................................................................................

....................................................................................................................

....................................................................................................................

....................................................................................................................

....................................................................................................................

....................................................................................................................

....................................................................................................................

....................................................................................................................

....................................................................................................................

....................................................................................................................

....................................................................................................................

....................................................................................................................

....................................................................................................................

....................................................................................................................

....................................................................................................................

....................................................................................................................

....................................................................................................................

....................................................................................................................

....................................................................................................................

....................................................................................................................

....................................................................................................................

....................................................................................................................

....................................................................................................................

....................................................................................................................

....................................................................................................................

....................................................................................................................

....................................................................................................................

....................................................................................................................

....................................................................................................................

....................................................................................................................

....................................................................................................................

....................................................................................................................

....................................................................................................................

....................................................................................................................

....................................................................................................................

....................................................................................................................

....................................................................................................................

....................................................................................................................

....................................................................................................................

....................................................................................................................

....................................................................................................................

....................................................................................................................

....................................................................................................................

....................................................................................................................

....................................................................................................................

....................................................................................................................

....................................................................................................................

....................................................................................................................

....................................................................................................................

....................................................................................................................

....................................................................................................................

....................................................................................................................

....................................................................................................................

....................................................................................................................

....................................................................................................................

....................................................................................................................

....................................................................................................................

....................................................................................................................

....................................................................................................................

....................................................................................................................

....................................................................................................................

....................................................................................................................

....................................................................................................................

....................................................................................................................

....................................................................................................................

....................................................................................................................

....................................................................................................................

....................................................................................................................

....................................................................................................................

....................................................................................................................

....................................................................................................................

....................................................................................................................

....................................................................................................................

....................................................................................................................

....................................................................................................................

....................................................................................................................

....................................................................................................................

....................................................................................................................

....................................................................................................................

....................................................................................................................

....................................................................................................................

....................................................................................................................

....................................................................................................................

....................................................................................................................

....................................................................................................................

....................................................................................................................

....................................................................................................................

....................................................................................................................

....................................................................................................................

....................................................................................................................

....................................................................................................................

....................................................................................................................

....................................................................................................................

....................................................................................................................

....................................................................................................................

....................................................................................................................

....................................................................................................................

....................................................................................................................

....................................................................................................................

....................................................................................................................

....................................................................................................................

....................................................................................................................

Page 35: Caderno 05 - Matematica

•M AT EM Á T I C A •Ori

enta

ções d

e E

stu

do

Page 36: Caderno 05 - Matematica
Page 37: Caderno 05 - Matematica

O RIENTAÇÕES D E E STUDO

Matemática

37

• ORIENTAÇÕES DE ESTUDO •

POLINÔMIOS

Atividade III → Exs.: 1, 2, 4, 8

PROBABILIDADE

Atividade III → Exs.: 2, 8, 9, 11

EQUAÇÕES POLINÔMIAIS

Atividade III → Exs.: 1, 2, 3, 15

NÚMEROS INTEIROS – DIVISIBILIDADE

Atividade III → Exs.: 1 a 3

MÓDULO DE UM NÚMERO REAL

Atividade III → Exs.: 1, 2, 3, 5

MATRIZES (2ª PARTE)

Atividade III → Ex.: 1

TRIGONOMETRIA (3ª PARTE)

Atividade III → Exs.: 1 e 9

GEOMETRIA DO ESPAÇO

Atividade III → Exs.: 1, 2, 4, 7, 9, 14, 15

Page 38: Caderno 05 - Matematica

38

• ORIENTAÇÕES DE ESTUDO •

O RIENTAÇÕES D E E STUDO

ANOTAÇÕES:

....................................................................................................................

....................................................................................................................

....................................................................................................................

....................................................................................................................

....................................................................................................................

....................................................................................................................

....................................................................................................................

....................................................................................................................

....................................................................................................................

....................................................................................................................

....................................................................................................................

....................................................................................................................

....................................................................................................................

....................................................................................................................

....................................................................................................................

....................................................................................................................

....................................................................................................................

....................................................................................................................

....................................................................................................................

....................................................................................................................

....................................................................................................................

....................................................................................................................

....................................................................................................................

....................................................................................................................

....................................................................................................................

....................................................................................................................

....................................................................................................................

....................................................................................................................

....................................................................................................................

....................................................................................................................

....................................................................................................................

....................................................................................................................

....................................................................................................................

....................................................................................................................

....................................................................................................................

....................................................................................................................

....................................................................................................................

....................................................................................................................

....................................................................................................................

....................................................................................................................

....................................................................................................................

....................................................................................................................

....................................................................................................................

....................................................................................................................

....................................................................................................................

....................................................................................................................

....................................................................................................................

....................................................................................................................

....................................................................................................................

....................................................................................................................

....................................................................................................................

....................................................................................................................

....................................................................................................................

....................................................................................................................

....................................................................................................................

....................................................................................................................

....................................................................................................................

....................................................................................................................

....................................................................................................................

....................................................................................................................

....................................................................................................................

....................................................................................................................

....................................................................................................................

....................................................................................................................

....................................................................................................................

....................................................................................................................

....................................................................................................................

....................................................................................................................

....................................................................................................................

....................................................................................................................

....................................................................................................................

....................................................................................................................

....................................................................................................................

....................................................................................................................

....................................................................................................................

....................................................................................................................

....................................................................................................................

....................................................................................................................

....................................................................................................................

....................................................................................................................

....................................................................................................................

....................................................................................................................

....................................................................................................................

....................................................................................................................

....................................................................................................................

....................................................................................................................

....................................................................................................................

....................................................................................................................

....................................................................................................................

....................................................................................................................

....................................................................................................................

....................................................................................................................

....................................................................................................................

....................................................................................................................

....................................................................................................................

....................................................................................................................

....................................................................................................................

....................................................................................................................

....................................................................................................................

....................................................................................................................

....................................................................................................................

....................................................................................................................

....................................................................................................................

....................................................................................................................

....................................................................................................................

....................................................................................................................

....................................................................................................................

....................................................................................................................

....................................................................................................................

....................................................................................................................

....................................................................................................................

....................................................................................................................